Top Banner
NCLEX RN Practice Questions 17 1. What’s the first intervention for a patient experiencing chest pain and an 5p02 of 89%? A. Administer morphine. B. Administer oxygen. C. Administer sublingual nitroglycerin. D. Obtain an electrocardiogram (ECC) 2. Which of the following signs and symptoms usually signifies rapid expansion and impending rupture of a abdominal aortic aneurysm? A. Abdominal pain. B. Absent pedal pulses. C. Chest pain. D. Lower back pain. 3. In which of the following types of cardiomyopathy does cardiac output remain normal? A. Dilated. B. Hypertrophic. C. Obliterative. D. Restrictive. 4. Which of the following interventions should be your first priority when treating a patient experiencin chest pain while walking? A. Have the patient sit down. B. Get the patient back to bed. C. Obtain an ECG. D. Administer sublingual nitroglycerin. 5. Which of the following positions would best aid breathing for a patient with acute pulmonary edema? A. Lying flat in bed. B. Left side-lying position. C. High Fowler’s position. D. Semi-Fowler’s position. 6. A pregnant woman arrives at the emergency department (ED) with abruptio placentae at 34 weeks’ gestation. She’s at risk for which of the following blood dyscrasias? A. Thrombocytopenia. B. Idiopathic thrombocytopenic purpura (ITP). C. Disseminated intravascular coagulation (DIC). D. Heparin-associated thrombosis and thrombocytopenia (HATT). 7. A 16-year-old patient involved in a motor vehicle accident arrives in the ED unconscious and severely hypotensive. He’s suspected to have several fractures of his pelvis and legs. Which of the following parenteral fluids is the best choice for his current condition? A. Fresh frozen plasma. B. 0.9% sodium chloride solution. C. Lactated Ringer’s solution. D. Packed red blood cells. 8. Corticosteroids are potent suppressors of the body’s inflammatory response. Which of the following conditions or actions do they suppress? A. Cushing syndrome. B. Pain receptors. C. Immune response.
33

NCLEX RN Practice Questions 17

Feb 09, 2016

Download

Documents

clarheena

NCLEX RN Practice Questions
v hhgfvygshbghsgbhgfvsgfuqagbhbhagfhgauuyruahjhfhhagfryaqtgyfghagjuyaweytgwyrfgygshfdhgshg
Welcome message from author
This document is posted to help you gain knowledge. Please leave a comment to let me know what you think about it! Share it to your friends and learn new things together.
Transcript
Page 1: NCLEX RN Practice Questions 17

NCLEX RN Practice Questions 17

1. What’s the first intervention for a patient experiencing chest pain and an 5p02 of 89%? 

A. Administer morphine. B. Administer oxygen. C. Administer sublingual nitroglycerin. D. Obtain an electrocardiogram (ECC) 

2. Which of the following signs and symptoms usually signifies rapid expansion and impending rupture of an abdominal aortic aneurysm? 

A. Abdominal pain. B. Absent pedal pulses. C. Chest pain. D. Lower back pain. 

3. In which of the following types of cardiomyopathy does cardiac output remain normal? 

A. Dilated. B. Hypertrophic. C. Obliterative. D. Restrictive. 

4. Which of the following interventions should be your first priority when treating a patient experiencing chest pain while walking? 

A. Have the patient sit down. B. Get the patient back to bed. C. Obtain an ECG. D. Administer sublingual nitroglycerin. 

5. Which of the following positions would best aid breathing for a patient with acute pulmonary edema? 

A. Lying flat in bed. B. Left side-lying position. C. High Fowler’s position. D. Semi-Fowler’s position. 

6. A pregnant woman arrives at the emergency department (ED) with abruptio placentae at 34 weeks’ gestation. She’s at risk for which of the following blood dyscrasias? 

A. Thrombocytopenia. B. Idiopathic thrombocytopenic purpura (ITP). C. Disseminated intravascular coagulation (DIC). D. Heparin-associated thrombosis and thrombocytopenia (HATT). 

7. A 16-year-old patient involved in a motor vehicle accident arrives in the ED unconscious and severely hypotensive. He’s suspected to have several fractures of his pelvis and legs. Which of the following parenteral fluids is the best choice for his current condition? 

A. Fresh frozen plasma. B. 0.9% sodium chloride solution. C. Lactated Ringer’s solution. D. Packed red blood cells. 

8. Corticosteroids are potent suppressors of the body’s inflammatory response. Which of the following conditions or actions do they suppress? 

A. Cushing syndrome. B. Pain receptors. C. Immune response. D. Neural transmission. 

9. A patient infected with human immunodeficiency virus (HIV) begins zidovudine therapy. Which of the following statements best describes this drug’s action? 

A. It destroys the outer wall of the virus and kills it. B. It interferes with viral replication. C. It stimulates the immune system. D. It promotes excretion of viral antibodies. 

10. A 20-year-old patient is being treated for pneumonia. He has a persistent cough and complains of severe pain on coughing. What could you tell him to help him reduce his discomfort? 

A. "Hold your cough as much as possible." 

Page 2: NCLEX RN Practice Questions 17

B. "Place the head of your bed flat to help with coughing." C. "Restrict fluids to help decrease the amount of sputum." D. "Splint your chest wall with a pillow for comfort." 

11. A 19-year-old patient comes to the ED with acute asthma. His respiratory rate is 44 breaths/minute, and he appears to be in acute respiratory distress. Which of the following actions should you take first? 

A. Take a full medical history. B. Give a bronchodilator by nebulizer. C. Apply a cardiac monitor to the patient. D. Provide emotional support for the patient. 

12. A firefighter who was involved in extinguishing a house fire is being treated for smoke inhalation. He develops severe hypoxia 48 hours after the incident, requiring intubation and mechanical ventilation. Which of the following conditions has he most likely developed? 

A. Acute respiratory distress syndrome (ARDS). B. Atelectasis. C. Bronchitis. D. Pneumonia. 

13. Which of the following measures best determines that a patient who had a pneumothorax no longer needs a chest tube? 

A. You see a lot of drainage from the chest tube. B. Arterial blood gas (ABG) levels are normal. C. The chest X-ray continues to show the lung is 35% deflated. D. The water-seal chamber doesn’t fluctuate when no suction is applied. 

14. Which of the following nursing interventions should you use to prevent footdrop and contractures in a patient recovering from a subdural hematoma? 

A. High-top sneakers. B. Low-dose heparin therapy. C. Physical therapy consultation. D. Sequential compressive device. 

15. Which of the following signs of increased intracranial pressure (ICP) would appear first after head trauma? 

A. Bradycardia. B. Large amounts of very dilute urine. C. Restlessness and confusion. D. Widened pulse pressure. 

16. When giving intravenous (I.V.) phenytoin, which of the following methods should you use? 

A. Use an in-line filter. B. Withhold other anticonvulsants. C. Mix the drug with saline solution only. D. Flush the I.V. catheter with dextrose solution. 

17. After surgical repair of a hip, which of the following positions is best for the patient’s legs and hips? 

A. Abduction. B. Adduction. C. Prone. D. Subluxated. 

18. Which of the following factors should be the primary focus of nursing management in a patient with acute pancreatitis? 

A. Nutrition management. B. Fluid and electrolyte balance. C. Management of hypoglycemia. D. Pain control. 

19. After a liver biopsy, place the patient in which of the following positions? 

A. Left side-lying, with the bed flat. B. Right side-lying, with the bed flat. C. Left side-lying, with the bed in semi-Fowler’s position. D. Right side-lying, with the bed in semi-Fowler’s position. 

20. Which of the following potentially serious complications could occur with therapy for hypothyroidism? 

A. Acute hemolytic reaction. B. Angina or cardiac arrhythmia. 

Page 3: NCLEX RN Practice Questions 17

C. Retinopathy. D. Thrombocytopenia. 

21. Adequate fluid replacement and vasopressin replacement are objectives of therapy for which of the following disease processes? 

A. Diabetes mellitus. B. Diabetes insipidus. C. Diabetic ketoacidosis. D. Syndrome of inappropriate antidiuretic hormone secretion (SIADH). 

22. Patients with Type 1 diabetes mellitus may require which of the following changes to their daily routine during periods of infection? 

A. No changes. B. Less insulin. C. More insulin. D. Oral diabetic agents. 

23. On a follow-up visit after having a vaginal hysterectomy, a 32-year-old patient has a decreased hematocrit level. Which of the following complications does this suggest? 

A. Hematoma. B. Hypovolemia. C. Infection. D. Pulmonary embolus (PE). 

24. A patient has partial-thickness burns to both legs and portions of his trunk. Which of the following I.V. fluids is given first? 

A. Albumin. B. D5W. C. Lactated Ringer’s solution. D. 0.9% sodium chloride solution with 2 mEq of potassium per 100 ml. 

25. Which of the following techniques is correct for obtaining a wound culture specimen from a surgical site? 

A. Thoroughly irrigate the wound before collecting the specimen. B. Use a sterile swab and wipe the crusty area around the outside of the wound. C. Gently roll a sterile swab from the center of the wound outward to collect drainage. D. Use a sterile swab to collect drainage from the dressing.

1.  B  Administering supplemental oxygen to the patient is the first priority.  Administer oxygen to increase SpO2 to greater than 90% to help prevent further cardiac damage.  Sublingual nitroglycerin and morphine are commonly administered after oxygen.

2.  D  Lower back pain results from expansion of the aneurysm.  The expansion applies pressure in the abdomen, and the pain is referred to the lower back.  Abdominal pain is the most common symptom resulting from impaired circulation.  Absent pedal pulses are a sign of no circulation and would occur after a ruptured aneurysm or in peripheral vascular disease.  Chest pain usually is associated with coronary artery or pulmonary disease.

3.  B  Cardiac output isn’t affected by hypertrophic cardiomyopathy because the size of the ventricle remains relatively unchanged.  Dilated cardiomyopathy, obliterative cardiomyopathy, and restrictive cardiomyopathy all decrease cardiac output.

4.  A The initial priority is to decrease oxygen consumption by sitting the patient down.  Administer sublingual nitroglycerin as you simultaneously do the ECG.  When the patient’s condition is stabilized, he can be returned to bed.

5.  C High Fowler’s position facilitates breathing by reducing venous return.  Lying flat and side-lying positions worsen breathing and increase the heart’s workload.

6.  C Abruptio placentae is a cause of DIC because it activates the clotting cascade after hemorrhage.  Thrombocytopenia results from decreased production of platelets.  ITP doesn’t have a definitive cause.  A patient with abruptio placentae wouldn’t get heparin and, as a result, wouldn’t be at risk for HATT.

7. D  In a trauma situation, the first blood product given is unmatched (0 negative) packed red blood cells.  Fresh frozen plasma often is used to replace clotting factors.  Lactated Ringer’s solution or 0.9% sodium chloride is used to increase volume and blood pressure, but too much of these crystalloids will dilute the blood and won’t improve oxygen-carrying capacity.

8.  C  Corticosteroids suppress eosinophils, lymphocytes, and natural-killer cells, inhibiting the natural inflammatory process in an infected or injured part of the body.  This helps resolve inflammation, stabilizes lysosomal membranes, decreases capillary permeability, and depresses phagocytosis of tissues by white blood cells, thus blocking the release of more inflammatory materials.  Excessive corticosteroid therapy can lead to Cushing syndrome.

9.  B  Zidovudine inhibits DNA synthesis in HIV, thus interfering with viral replication.  The drug doesn’t destroy the viral wall, stimulate the immune system, or promote HIV antibody excretion.

10. D  Showing this patient how to splint his chest wall will help decrease discomfort when coughing.  Holding in his coughs will only increase his pain.  Placing the head of the bed flat may increase the frequency of his cough and his work of breathing.  Increasing fluid intake will help thin his secretions, making it easier for him to clear them.

11. B  The patient having an acute asthma attack needs more oxygen delivered to his lungs and body.  Nebulized bronchodilators open airways and increase the amount of oxygen delivered.  The patient may not need cardiac monitoring because he’s only 19 years old, unless he has a medical history of cardiac problems.

12. A  Severe hypoxia after smoke inhalation typically is related to ARDS.  The other choices aren’t typically associated with smoke inhalation.

13. D  The chest tube isn’t removed until the patient’s lung has adequately reexpanded and is expected to stay that way.  One indication of reexpansion is the cessation of fluctuation in the water-seal chamber when suction isn’t applied.  The chest X-ray should show that the lung is reexpanded.  Drainage should be minimal before the chest tube is removed.  An ABG test isn’t necessary if clinical assessment criteria are met.

Page 4: NCLEX RN Practice Questions 17

14. A  High-top sneakers are used to prevent footdrop and contractures in patients with neurologic conditions.  A consult with physical therapy is important to prevent footdrop, but you can use high-top sneakers independently.

15. C  The earliest sign of increased ICP is a change in mental status.  Bradycardia and widened pulse pressure occur later.  The patient may void a lot of very dilute urine if his posterior pituitary is damaged.

16. C  Phenytoin is compatible only with saline solutions; dextrose causes an insoluble precipitate to form.  You needn’t withhold additional anticonvulsants or use an in-line filter.

17. A  After surgical repair of the hip, keep the legs and hips abducted to stabilize the prosthesis in the acetabulum.

18. B  Acute pancreatitis is commonly associated with fluid isolation and accumulation in the bowel secondary to ileus or peripancreatic edema.  Fluid and electrolyte loss from vomiting is a major concern.  Therefore, your priority is to manage hypovolemia and restore electrolyte balance.  Pain control and nutrition also are important.  Patients are at risk for hyperglycemia, not hypoglycemia.

19. B  Positioning the patient on his right side with the bed flat will splint the biopsy site and minimize bleeding.  The other positions won’t do this and may cause more bleeding at the site or internally.

20. B  Precipitation of angina or cardiac arrhythmia is a potentially serious complication of hypothyroidism treatment.  Acute hemolytic reaction is a complication of blood transfusions.  Retinopathy typically is a complication of diabetes mellitus.  Thrombocytopenia doesn’t result from treating hypothyroidism.

21. B  Maintaining adequate fluid and replacing vasopressin are the main objectives in treating diabetes insipidus.  An excess of antidiuretic hormone leads to SIADH, causing the patient to retain fluid.  Diabetic ketoacidosis is a result of severe insulin insufficiency.

22. C  During periods of infection or illness, patients with Type 1 diabetes may need even more insulin to compensate for increased blood glucose levels.

23. A  A decreased hematocrit level is a sign of hematoma, a delayed complication of abdominal and vaginal hysterectomy.  Symptoms of hypovolemia include increased hematocrit and hemoglobin values.  Symptoms of a PE include dyspnea, chest pain, cough, hemoptysis, restlessness, and signs of shock.

24. C  Lactated Ringer’s solution replaces lost sodium and corrects metabolic acidosis, both of which commonly occur following a burn.  Albumin is used as adjunct therapy, not primary fluid replacement.  Dextrose isn’t given to burn patients during the first 24 hours because it can cause pseudodiabetes.  The patient is hyperkalemic from the potassium shift from the intracellular space to the plasma, so potassium would be detrimental.

25. C  Rolling a swab from the center outward is the right way to obtain a culture specimen from a wound.  Irrigating the wound washes away drainage, debris, and many of the colonizing or infecting microorganisms.  The outside of the wound and the dressing may be colonized with microorganisms that haven’t affected the wound, so specimens from these sites could give inaccurate results.

. A patient with cirrhosis who is being treated with spironolactone (Aldactone) and furosemide (Lasix) has a serum sodium level of 135 mEq/L (135 mmol/L) and serum potassium 3.2 mEq/L (3.2 mmol/L). Before notifying the health care provider, the nurse

should

a. administer the furosemide and withhold the spironolactone.b. give both drugs as scheduled.c. administer the spironolactone.

d. withhold both drugs until talking with the health care provider.C

Rationale: Spironolactone is a potassium-sparing diuretic and will help to increase the patient's potassium level. 

The nurse does not need to talk with the doctor before giving the spironolactone, although the health care provider should be notified about the low potassium value. 

The furosemide will further decrease the patient's potassium level and should be held until the nurse talks with the health care provider.

Terms Definitions

. A patient with cirrhosis who is being treated with spironolactone (Aldactone) and furosemide (Lasix) has a serum sodium level

of 135 mEq/L (135 mmol/L) and serum potassium 3.2 mEq/L (3.2 mmol/L). Before

notifying the health care provider, the nurse should

a. administer the furosemide and withhold the spironolactone.

b. give both drugs as scheduled.c. administer the spironolactone.

d. withhold both drugs until talking with the health care provider.

C

Rationale: Spironolactone is a potassium-sparing diuretic and will help to increase the

patient's potassium level. 

The nurse does not need to talk with the doctor before giving the spironolactone,

although the health care provider should be notified about the low potassium value. 

The furosemide will further decrease the patient's potassium level and should be held

until the nurse talks with the health care provider.

A 32-year-old patient has early alcoholic cirrhosis diagnosed by a liver biopsy. When

planning patient teaching, the priority information for the nurse to include is the

need for

a. vitamin B supplements.b. abstinence from alcohol.

c. maintenance of a nutritious diet.d. long-term, low-dose corticosteroids.

BRationale: The disease progression can be

stopped or reversed by alcohol abstinence. 

The other interventions may be used when cirrhosis becomes more severe to decrease symptoms or complications, but the priority for this patient is to stop the progression of

the disease.

A 37-year-old forklift operator presents with shakiness, sweating, anxiety, and

palpitations and tells the nurse he has type 1 diabetes mellitus. Which of the follow

Correct answer: C

Because the client is awake and complaining of symptoms, the nurse should

Page 5: NCLEX RN Practice Questions 17

actions should the nurse do first?

A. Inject 1 mg of glucagon subcutaneously. B. Administer 50 mL of 50% glucose I.V. 

C. Give 4 to 6 oz (118 to 177 mL) of orange juice. 

D. Give the client four to six glucose tablets.

first give him 15 grams of carbohydrate to treat hypoglycemia. This could be 4 to 6 oz

of fruit juice, five to six hard candies such as Lifesavers, or 1 tablespoon of sugar. When a

client has worsening symptoms of hypoglycemia or is unconscious, treatment includes 1 mg of glucagon subcutaneously

or intramuscularly, or 50 mL of 50% glucose I.V. The nurse may also give two to three

glucose tablets for a hypoglycemic reaction.

A 60 year old patient has an abrupt onset of anorexia, nausea and vomiting,

hepatomegaly, and abnormal liver function studies. Serologic testing is negative for

viral causes of hepatitis. During assessment of the patient, it is most important for the nurse to question the patient regarding

A. any prior exposure to people with jaundice

B. the use of all prescription and OTC (over the counter) medications

C. treatment of chronic diseases with corticosteriods

D. exposure to children recently immunized for hepatitis B

A and D assess for exposure to hepatitis. Hepatitis was ruled out this is inappropriate.

C is incorrect because corticosteroids do not commonly cause liver disease

B is correct because overdose of medications can cause liver disease.

A 63-year-old patient is newly diagnosed with type 2 diabetes. When developing an

education plan, the nurse's first action should be to

a. assess the patient's perception of what it means to have type 2 diabetes.

b. demonstrate how to check glucose using capillary blood glucose monitoring.

c. ask the patient's family to participate in the diabetes education program.

d. discuss the need for the patient to actively participate in diabetes

management.

A

Rationale: Before planning education, the nurse should assess the patient's interest in

and ability to self-manage the diabetes. 

After assessing the patient, the other nursing actions may be appropriate, but

planning needs to be individualized to each patient.

A 1200-calorie diet and exercise are prescribed for a patient with newly

diagnosed type 2 diabetes. The patient tells the nurse, "I hate to exercise! Can't I just follow the diet to keep my glucose under

control?" The nurse teaches the patient that the major purpose of exercise for diabetics

is to

a. increase energy and sense of well-being, which will help with body image.

b. facilitate weight loss, which will decrease peripheral insulin resistance.

c. improve cardiovascular endurance, which is important for diabetics.

d. set a successful pattern, which will help in making other needed changes.

b. facilitate weight loss, which will decrease peripheral insulin resistance.

Rationale: Exercise is essential to decrease insulin resistance and improve blood

glucose control. Increased energy, improved cardiovascular endurance, and setting a

pattern of success are secondary benefits of exercise, but they are not the major reason.

A client has just had surgery for colon cancer. Which of the following disorders

might the client develop?

a. Peritonitisb. Diverticulosis

c. Partial bowel obstructiond. Complete bowel obstruction

a. Peritonitis

Bowel spillage could occur during surgery, resulting in peritonitis. Complete or partial bowel obstruction may occur before bowel resection. Diverticulosis doesn't result from

surgery or colon cancer.

A client has recently been diagnosed with Type I diabetes and asks the nurse for help formulating a nutrition plan. Which of the

following recommendations would the nurse make to help the client increase calorie

consumption to offset absorption problems?

a. Eating small meals with two or three snacks may be more helpful in maintaining blood glucose levels than three large meals.

b. Eat small meals with two or three snacks throughout the day to keep blood glucose

levels steady

AEating small meals with two or three snacks may be more helpful in maintaining blood

glucose levels than three large meals.

Page 6: NCLEX RN Practice Questions 17

c. Increase consumption of simple carbohydrates

d. Skip meals to help lose weight

A client is admitted to the medical-surgical floor with a diagnosis of acute pancreatitis. His blood pressure is 136/76 mm Hg, pulse

96 beats/minute, respirations 22 breaths/minute, temperature 99°F (38.3°C),

and he has been experiencing severe vomiting for 24 hours. His past medical

history reveals hyperlipidemia and alcohol abuse. The physician prescribes a

nasogastric (NG) tube for the client. Which of the following is the primary purpose for

insertion of the NG tube?

A. Empty the stomach of fluids and gas to relieve vomiting. 

B. Prevent spasms at the sphincter of Oddi. C. Prevent air from forming in the small and

large intestines. D. Remove bile from the gallbladder.

Correct answer: A

An NG tube is no longer routinely inserted to treat pancreatitis, but if the client has

protracted vomiting, the NG tube is inserted to drain fluids and gas and relieve vomiting. 

An NG tube doesn't prevent spasms at the sphincter of Oddi (a valve in the duodenum that controls the flow of digestive enzymes) or prevent air from forming in the small and

large intestine. 

The common bile duct connects to the pancreas and the gall bladder, and a T tube

rather than an NG tube would be used to collect bile drainage from the common bile

duct.

A client is brought to the emergency room in an unresponsive state, and a diagnosis of

hyperglycemic hyperosmolar nonketotic syndrome is made. The nurse would

immediately prepare to initiate which of the following anticipated physician's orders?

a) endotracheal intubationb) 100 units of NPH insulin

c) intravenous infusion of normal salined) intravenous infusion of sodium

bicarbonate

c) intravenous infusion of normal saline

The primary goal of treatment in hyperglycemic hyperosmolar nonketotic

syndrome (HHNS) is to rehydrate the client to restore fluid volume and to correct

electrolyte deficiency. 

Intravenous fluid replacement is similar to that administered in diabetic ketoacidosis

(DKA) and begins with IV infusion of normal saline. 

Regular insulin, not NPH insulin, would be administered. The use of sodium

bicarbonate to correct acidosis is avoided because it can precipitate a further drop in

serum potassium levels. Intubation and mechanical ventilation are not required to

treat HHNS.

A client who recently underwent cranial surgery develops syndrome of inappropriate antidiuretic hormone (SIADH). Which of the

following symptoms should the nurse anticipate?

A. Edema and weight gain. B. Excessive urinary output. 

C. Fluid loss and dehydration. D. Low urine specific gravity.

Correct answer: A

Syndrome of inappropriate antidiuretic hormone (SIADH) results in an abnormally

high release of antidiuretic hormone, which causes water retention as serum sodium levels fall, leading to edema and weight gain. Because of fluid retention, urine

output is low. 

Fluid is restricted to prevent fluid overload rather than replaced. As the urine becomes

more concentrated, the specific gravity increases. Other symptoms include nausea, vomiting, seizures, altered mentation, and

coma. 

SIADH is most common with diseases of the hypothalamus but can also occur with heart failure, Guillain-Barré syndrome, meningitis, encephalitis, head trauma, or brain tumors.

It may also be triggered by medications.

A client with cirrhosis of the liver develops ascites. Which of the following orders would

the nurse expect?

A. Restrict fluid to 1000 mL per day. B. Ambulate 100 ft. three times per day. 

C. High-sodium diet. D. Maalox 30 ml P.O. BID.

Correct answer: A

Fluid restriction is a primary treatment for ascites. Restricting fluids decreases the

amount of fluid present in the body, thereby decreasing the fluid that accumulates in the

peritoneal space. 

A high sodium diet would increase fluid retention. Physical activities are usually

restricted until ascites is relieved. 

Loop diuretics (such as furosemide) are usually ordered, and Maalox® (a bismuth

subsalicylate) may interfere with the action of the diuretics.

A college student who has type 1 diabetes d. check glucose level before, during, and

Page 7: NCLEX RN Practice Questions 17

normally walks each evening as part of an exercise regimen. The student now plans to

take a swimming class every day at 1:00 PM. The clinic nurse teaches the patient to

a. delay eating the noon meal until after the swimming class.

b. increase the morning dose of neutral protamine Hagedorn (NPH) insulin on days

of the swimming class.c. time the morning insulin injection so that

the peak occurs while swimming.d. check glucose level before, during, and

after swimming.

after swimming.

Rationale: The change in exercise will affect blood glucose, and the patient will need to monitor glucose carefully to determine the

need for changes in diet and insulin administration. 

Because exercise tends to decrease blood glucose, patients are advised to eat before

exercising. 

Increasing the morning NPH or timing the insulin to peak during exercise may lead to hypoglycemia, especially with the increased

exercise.

A diabetic patient is admitted with ketoacidosis and the health care provider writes all of the following orders. Which order should the nurse implement first?

a. Start an infusion of regular insulin at 50 U/hr.

b. Give sodium bicarbonate 50 mEq IV push.c. Infuse 1 liter of normal saline per hour.

d. Administer regular IV insulin 30 U.

CRationale: The most urgent patient problem

is the hypovolemia associated with DKA, and the priority is to infuse IV fluids. 

The other actions can be accomplished after the infusion of normal saline is initiated.

A diabetic patient is started on intensive insulin therapy. The nurse will plan to teach the patient about mealtime coverage using

_____ insulin.

a. NPH b. lispro 

c. detemir d. glargine

BRationale: Rapid or short acting insulin is used for mealtime coverage for patients receiving intensive insulin therapy. NPH, glargine, or detemir will be used as the

basal insulin.

A diagnosis of hyperglycemic hyperosmolar nonketotic coma (HHNC) is made for a

patient with type 2 diabetes who is brought to the emergency department in an

unresponsive state. The nurse will anticipate the need to

a. administer glargine (Lantus) insulin.b. initiate oxygen by nasal cannula.c. insert a large-bore IV catheter.d. give 50% dextrose as a bolus.

C

Rationale: HHNC is initially treated with large volumes of IV fluids to correct

hypovolemia. 

Regular insulin is administered, not a long-acting insulin. There is no indication that the patient requires oxygen. Dextrose solutions will increase the patient's blood glucose and

would be contraindicated.

A frail elderly patient with a diagnosis of type 2 diabetes mellitus has been ill with pneumonia. The client's intake has been

very poor, and she is admitted to the hospital for observation and management

as needed. What is the most likely problem with this patient?

a. Insulin resistance has developed.b. Diabetic ketoacidosis is occurring.

c. Hypoglycemia unawareness is developing.

d. Hyperglycemic hyperosmolar non-ketotic coma

d. Hyperglycemic hyperosmolar non-ketotic coma

Illness, especially with the frail elderly patient whose appetite is poor, can result in

dehydration and HHNC. 

Insulin resistance usually is indicated by a daily insulin requirement of 200 units or

more. 

Diabetic ketoacidosis, an acute metabolic condition, usually is caused by absent or markedly decreased amounts of insulin.

A health care provider who has not been immunized for hepatitis B is exposed to the

hepatitis B virus (HBV) through a needle stick from an infected patient. The infection

control nurse informs the individual that treatment for the exposure should include

a. baseline hepatitis B antibody testing now and in 2 months.

b. active immunization with hepatitis B vaccine.

c. hepatitis B immune globulin (HBIG) injection.

d. both the hepatitis B vaccine and HBIG injection.

D

Rationale: The recommended treatment for exposure to hepatitis B in unvaccinated

individuals is to receive both HBIG and the hepatitis B vaccine, which would provide

temporary passive immunity and promote active immunity. 

Antibody testing may also be done, but this would not provide protection from the

exposure.

A homeless patient with severe anorexia, fatigue, jaundice, and hepatomegaly is

diagnosed with viral hepatitis and has just been admitted to the hospital. In planning

ARationale: The highest priority outcome is to

maintain nutrition because adequate nutrition is needed for hepatocyte

Page 8: NCLEX RN Practice Questions 17

care for the patient, the nurse assigns the highest priority to the patient outcome of

a. maintaining adequate nutrition.b. establishing a stable home environment.

c. increasing activity level.d. identifying the source of exposure to

hepatitis.

regeneration. Finding a home for the patient and identifying the source of the infection

would be appropriate activities, but they do not have as high a priority as having

adequate nutrition. Although the patient's activity level will be gradually increased,

rest is indicated during the acute phase of hepatitis.

A hospitalized diabetic patient receives 12 U of regular insulin mixed with 34 U of NPH

insulin at 7:00 AM. The patient is away from the nursing unit for diagnostic testing at

noon, when lunch trays are distributed. The most appropriate action by the nurse is to

a. save the lunch tray to be provided upon the patient's return to the unit.

b. call the diagnostic testing area and ask that a 5% dextrose IV be started.

c. ensure that the patient drinks a glass of milk or orange juice at noon in the

diagnostic testing area.d. request that the patient be returned to the unit to eat lunch if testing will not be

completed promptly.

D

Rationale: Consistency for mealtimes assists with regulation of blood glucose, so the best option is for the patient to have lunch at the

usual time. 

Waiting to eat until after the procedure is likely to cause hypoglycemia. 

Administration of an IV solution is unnecessarily invasive for the patient. A

glass of milk or juice will keep the patient from becoming hypoglycemic but will cause a rapid rise in blood glucose because of the rapid absorption of the simple carbohydrate

in these items.

A newly diagnosed type 1 diabetic patient likes to run 3 miles several mornings a

week. Which teaching will the nurse implement about exercise for this patient?

a. "You should not take the morning NPH insulin before you run."

b. "Plan to eat breakfast about an hour before your run."

c. "Afternoon running is less likely to cause hypoglycemia."

d. "You may want to run a little farther if your glucose is very high."

B

Rationale: Blood sugar increases after meals, so this will be the best time to

exercise. 

NPH insulin will not peak until mid-afternoon and is safe to take before a morning run. 

Running can be done in either the morning or afternoon. If the glucose is very elevated,

the patient should postpone the run.

A nurse cares for a client following a liver biopsy. Which nursing care plan reflects

proper care?

a. Position in a dorsal recumbent position, with one pillow under the head

b. Bed rest for 24 hours, with a pressure dressing over the biopsy site

c. Position to a right side-lying position, with a pillow under the biopsy site

d. Neurological checks of lower extremities every hour

c. Position to a right side-lying position, with a pillow under the biopsy site

Positioning the client in a right side-lying position with a pillow under the biopsy site

reflects proper care. 

Answer 1 does not permit the necessary pressure applied to the biopsy site. B

ed rest is only required for several hours. 

There is no reason to do neurological checks.

A patient hospitalized with possible acute pancreatitis has severe abdominal pain and

nausea and vomiting. The nurse would expect the diagnosis to be confirmed with laboratory testing that reveals elevated

serum

a. calcium.b. bilirubin.c. amylase.

d. potassium.

CRationale: Amylase is elevated early in

acute pancreatitis.

Changes in bilirubin, calcium, and potassium levels are not diagnostic for pancreatitis.

A patient in the outpatient clinic has positive serologic testing for anti-HCV. Which action

by the nurse is appropriate?

a. Schedule the patient for HCV genotype testing.

b. Teach the patient that the HCV will resolve in 2 to 4 months.

c. Administer immune globulin and the HCV vaccine.

d. Instruct the patient on self-administration of -interferon.

ARationale: Genotyping of HCV has an

important role in managing treatment and is done before drug therapy with -interferon or other medications is started. HCV has a

high percentage of conversion to the chronic state so the nurse should not teach the patient that the HCV will resolve in 2 to

4 months. Immune globulin or vaccine is not available for HCV.

A patient is admitted to the hospital with a diagnosis of primary hyperparathyroidism. A

nurse checking the patient's lab results would expect which of the following changes

Answer: A

The parathyroid glands regulate the calcium level in the blood. 

Page 9: NCLEX RN Practice Questions 17

in laboratory findings?

A. Elevated serum calcium. B. Low serum parathyroid hormone (PTH). 

C. Elevated serum vitamin D. D. Low urine calcium.

In hyperparathyroidism, the serum calcium level will be elevated. 

Parathyroid hormone levels may be high or normal but not low. 

The body will lower the level of vitamin D in an attempt to lower calcium. Urine calcium may be elevated, with calcium spilling over from elevated serum levels. This may cause

renal stones.

A patient is admitted with an abrupt onset of jaundice, nausea and vomiting,

hepatomegaly, and abnormal liver function studies. Serologic testing is negative for

viral causes of hepatitis. Which question by the nurse is most appropriate?

a. "Have you been around anyone with jaundice?"

b. "Do you use any prescription or over-the-counter (OTC) drugs?"

c. "Are you taking corticosteroids for any reason?"

d. "Is there any history of IV drug use?"

B

Rationale: The patient's symptoms, lack of antibodies for hepatitis, and the ABRUPT

onset of symptoms suggest toxic hepatitis, which can be caused by commonly used

OTC drugs such as acetaminophen (Tylenol). 

Exposure to a jaundiced individual and a history of IV drug use are risk factors for

VIRAL hepatitis. 

Corticosteroid use does not cause the symptoms listed.

A patient newly diagnosed with Type I DM is being seen by the home health nurse. The doctors orders include: 1200 calorie ADA

diet, 15 units NPH insulin before breakfast, and check blood sugar qid. When the nurse

visits the patient at 5 pm, the nurse observes the man performing blood sugar analysis. The result is 50 mg/dL. The nurse

would expect the patient to be

a. confused with cold, clammy skin an pulse of 110

b. lethargic with hot dry dkin and rapid deep respirations

c. alert and cooperative with BP of 130/80 and respirations of 12

d. short of breath, with distended neck veins and bounding pulse of 96.

a. confused with cold, clammy skin an pulse of 110

hypoglycemia

A patient received 6 units of REGULAR INSULIN 3 hours ago. The nurse would be MOST concerned if which of the following

was observed?

a. kussmaul respirations and diaphoresisb. anorexia and lethargy

c. diaphoresis and tremblingd. headache and polyuria

c. diaphoresis and trembling

indicates hypoglycemia

A patient recovering from DKA asks the nurse how acidosis occurs. The best

response by the nurse is that

a. insufficient insulin leads to cellular starvation, and as cells rupture they release

organic acids into the blood.

b. when an insulin deficit causes hyperglycemia, then proteins are

deaminated by the liver, causing acidic by-products.

c. excess glucose in the blood is metabolized by the liver into acetone, which

is acidic.

d. an insulin deficit promotes metabolism of fat stores, which produces large amounts of

acidic ketones.

D

Rationale: Ketoacidosis is caused by the breakdown of fat stores when glucose is not available for intracellular metabolism. The

other responses are inaccurate.

A patient screened for diabetes at a clinic has a fasting plasma glucose level of 120 mg/dl (6.7 mmol/L). The nurse will plan to

teach the patient about

a. use of low doses of regular insulin.b. self-monitoring of blood glucose.

D

Rationale: The patient's impaired fasting glucose indicates pre-diabetes and the

patient should be counseled about LIFESTYLE CHANGES to prevent the development of type 2 diabetes. 

Page 10: NCLEX RN Practice Questions 17

c. oral hypoglycemic medications.d. maintenance of a healthy weight. The patient with prediabetes does not

require insulin or the oral hypoglycemics for glucose control and does not need to self-

monitor blood glucose.

A patient using a split mixed-dose insulin regimen asks the nurse about the use of

intensive insulin therapy to achieve tighter glucose control. The nurse should teach the

patient that

a. intensive insulin therapy requires three or more injections a day in addition to an injection of a basal long-acting insulin.

b. intensive insulin therapy is indicated only for newly diagnosed type 1 diabetics who

have never experienced ketoacidosis.c. studies have shown that intensive insulin therapy is most effective in preventing the macrovascular complications characteristic

of type 2 diabetes.d. an insulin pump provides the best

glucose control and requires about the same amount of attention as intensive insulin

therapy.

ARationale: Patients using intensive insulin

therapy must check their glucose level four to six times daily and administer insulin

accordingly. A previous episode of ketoacidosis is not a contraindication for

intensive insulin therapy. 

Intensive insulin therapy is not confined to type 2 diabetics and would prevent microvascular changes as well as

macrovascular changes.

Intensive insulin therapy and an insulin pump are comparable in glucose control.

A patient who has just been diagnosed with type 2 diabetes is 5 ft 4 in (160 cm) tall and

weighs 182 pounds (82 kg). A nursing diagnosis of imbalanced nutrition: more than body requirements is developed.

Which patient outcome is most important for this patient?

a. The patient will have a diet and exercise plan that results in weight loss.

b. The patient will state the reasons for eliminating simple sugars in the diet.c. The patient will have a glycosylated

hemoglobin level of less than 7%.d. The patient will choose a diet that

distributes calories throughout the day.

CRationale: The complications of diabetes are related to elevated blood glucose, and the

most important patient outcome is the reduction of glucose to near-normal levels. The other outcomes are also appropriate

but are not as high in priority.

A patient who is admitted with acute hepatic encephalopathy and ascites

receives instructions about appropriate diet. The nurse determines that the teaching has been effective when the patient's choice of

foods from the menu includes

a. an omelet with cheese and mushrooms and milk.

b. pancakes with butter and honey and orange juice.

c. baked beans with ham, cornbread, potatoes, and coffee.

d. baked chicken with french-fries, low-fiber bread, and tea.

b. pancakes with butter and honey and orange juice.

BRationale: The patient with acute hepatic

encephalopathy is placed on a LOW-protein diet to decrease ammonia levels. 

The other choices are all higher in protein and would not be as appropriate for this

patient. 

In addition, the patient's ascites indicate that a low-sodium diet is needed and the

other choices are all high in sodium.

A patient with acute pancreatitis has a nasogastric (NG) tube to suction and is NPO.

The nurse explains to the patient that the major purpose of this treatment is

a. control of fluid and electrolyte imbalance.b. relief from nausea and vomiting.c. reduction of pancreatic enzymes.

d. removal of the precipitating irritants.

c. reduction of pancreatic enzymes.

Rationale: Pancreatic enzymes are released when the patient eats. NG suction and NPO

status decrease the release of these enzymes. 

Fluid and electrolyte imbalances will be caused by NG suction and require that the patient receive IV fluids to prevent this. 

The patient's nausea and vomiting may decrease, but this is not the major reason

for these treatments. The pancreatic enzymes that precipitate the pancreatitis

are not removed by NG suction.

A patient with cancer of the liver has severe ascites, and the health care provider plans a paracentesis to relieve the fluid pressure on the diaphragm. To prepare the patient for

the procedure, the nurse

a. asks the patient to empty the bladder.b. positions the patient on the right side.

c. obtains informed consent for the procedure.

A

Rationale: The patient should empty the bladder to decrease the risk of bladder

perforation during the procedure.

The patient would be positioned in Fowler's position and would not be able to lie flat

without compromising breathing. 

Page 11: NCLEX RN Practice Questions 17

d. assists the patient to lie flat in bed. The health care provider is responsible for obtaining informed consent.

A patient with cirrhosis has 4+ pitting edema of the feet and legs and massive ascites. The data indicate that it is most important for the nurse to monitor the

patient's

a. temperature.b. albumin level.c. hemoglobin.d. activity level.

B

Rationale: The low oncotic pressure caused by hypoalbuminemia is a major

pathophysiologic factor in the development of ascites and edema. The other parameters should also be monitored, but they are not contributing factors to the patient's current

symptoms.

A patient with cirrhosis has a massive hemorrhage from esophageal varices. In planning care for the patient, the nurse gives the highest priority to the goal of

a. controlling bleeding.b. maintenance of the airway.

c. maintenance of fluid volume.d. relieving the patient's anxiety.

B

Rationale: Maintaining gas exchange has the highest priority because oxygenation is

essential for life.

The airway is compromised by the bleeding in the esophagus and aspiration easily

occurs. 

The other goals would also be important for this patient, but they are not as high a

priority as airway maintenance.

A patient with cirrhosis is being treated with spironolactone (Aldactone) tid and

furosemide (Lasix) bid. The patient's most recent laboratory results indicate a serum sodium of 134 mEq/L (134 mmol/L) and a

serum potassium of 3.2 mEq/L (3.2 mmol/L). Before notifying the physician, the nurse

should

A. administer only the furosemideB. administer both drugs as ordered

C. administer only the spironolactoneD. Withhold the furosemide and

spironolactone

C. administer only the spironolactone

The potassium level is dangerously low. Lasix is potassium depleting, while

spironolactone is potassium sparing. You would hold the Lasix and call the physician.

This is a good NCLEX question that integrates this course with pharmacology.

A patient with severe cirrhosis has an episode of bleeding esophageal varices. To

detect possible complications of the bleeding episode, it is most important for

the nurse to monitor

a. prothrombin time.b. bilirubin levels.c. ammonia levels.d. potassium levels.

C

Rationale: The blood in the GI tract will be absorbed as protein and may result in an increase in ammonia level since the liver

cannot metabolize protein well. 

The prothrombin time, bilirubin, and potassium levels should also be monitored,

but these will not be affected by the bleeding episode.

A patient with type 1 diabetes has been using self-monitoring of blood glucose

(SMBG) as part of diabetes management. During evaluation of the patient's technique

of SMBG, the nurse identifies a need for additional teaching when the patient

a. chooses a puncture site in the center of the finger pad.

b. washes the puncture site using soap and water.

c. says the result of 130 mg indicates good blood sugar control.

d. hangs the arm down for a minute before puncturing the site.

A

Rationale: The patient is taught to choose a puncture site at the side of the finger pad.

The other patient actions indicate that teaching has been effective.

A patient with type 1 diabetes has received diet instruction as part of the treatment plan. The nurse determines a need for

additional instruction when the patient says,

a. "I may have an occasional alcoholic drink if I include it in my meal plan."

b. "I will need a bedtime snack because I take an evening dose of NPH insulin."

c. "I will eat meals as scheduled, even if I am not hungry, to prevent hypoglycemia."d. "I may eat whatever I want, as long as I use enough insulin to cover the calories."

D

Rationale: Most patients with type 1 diabetes need to plan diet choices very

carefully. 

Patients who are using intensified insulin therapy have considerable flexibility in diet

choices but still should restrict dietary intake of items such as fat, protein, and

alcohol. 

The other patient statements are correct and indicate good understanding of the diet

Page 12: NCLEX RN Practice Questions 17

instruction.

A patient with type 1 diabetes who uses glargine (Lantus) and lispro (Humalog)

insulin develops a sore throat, cough, and fever. When the patient calls the clinic to report the symptoms and a blood glucose level of 210 mg/dl, the nurse advises the

patient to 

a. use only the lispro insulin until the symptoms of infection are resolved.

b. monitor blood glucose every 4 hours and notify the clinic if it continues to rise.

c. decrease intake of carbohydrates until glycosylated hemoglobin is less than 7%.d. limit intake to non-calorie-containing

liquids until the glucose is within the usual range.

B

Rationale: Infection and other stressors increase blood glucose levels and the patient will need to test blood glucose

frequently, treat elevations appropriately with insulin, and call the health care

provider if glucose levels continue to be elevated. 

Discontinuing the glargine will contribute to hyperglycemia and may lead to DKA. 

Decreasing carbohydrate or caloric intake is not appropriate as the patient will need

more calories when ill. 

Glycosylated hemoglobins are not used to test for short-term alterations in blood

glucose.

A patient with type 2 diabetes has sensory neuropathy of the feet and legs and

peripheral vascular disease evidenced by decreased peripheral pulses and dependent

rubor. The nurse teaches the patient that

a. the feet should be soaked in warm water on a daily basis.

b. flat-soled leather shoes are the best choice to protect the feet from injury.

c. heating pads should always be set at a very low temperature.

d. over-the-counter (OTC) callus remover may be used to remove callus and prevent

pressure.

B

Rationale: The patient is taught to avoid high heels and that leather shoes are

preferred. 

The feet should be washed, but not soaked, in warm water daily. 

Heating pad use should be avoided. 

Commercial callus and corn removers should be avoided; the patient should see a

specialist to treat these problems.

A patient with type 2 diabetes is scheduled for an outpatient coronary arteriogram.

Which information obtained by the nurse when admitting the patient indicates a need

for a change in the patient's regimen?

a. The patient's most recent hemoglobin A1C was 6%.

b. The patient takes metformin (Glucophage) every morning.

c. The patient uses captopril (Capoten) for hypertension.

d. The patient's admission blood glucose is 128 mg/dl.

BRationale: To avoid lactic acidosis,

metformin should not be used for 48 hours after IV contrast media are administered. 

The other patient data indicate that the patient is managing the diabetes

appropriately.

A patient with type 2 diabetes that is controlled with diet and metformin

(Glucophage) also has severe rheumatoid arthritis (RA). During an acute exacerbation

of the patient's arthritis, the health care provider prescribes prednisone (Deltasone)

to control inflammation. The nurse will anticipate that the patient may 

a. require administration of insulin while taking prednisone.

b. develop acute hypoglycemia during the RA exacerbation.

c. have rashes caused by metformin-prednisone interactions.

d. need a diet higher in calories while receiving prednisone.

A

Rationale: Glucose levels increase when patients are taking CORTICOsteroids, and insulin may be required to control blood

glucose. 

Hypoglycemia is not a complication of RA exacerbation or prednisone use.

Rashes are not an adverse effect caused by taking metformin and prednisone

simultaneously. 

The patient is likely to have an increased appetite when taking prednisone, but it will be important to avoid weight gain for the

patient with RA.

A program of weight loss and exercise is recommended for a patient with impaired fasting glucose (IFG). When teaching the

patient about the reason for these lifestyle changes, the nurse will tell the patient that

a. the high insulin levels associated with this syndrome damage the lining of blood vessels, leading to vascular disease.

b. although the fasting plasma glucose levels do not currently indicate diabetes, the

glycosylated hemoglobin will be elevated.c. the liver is producing excessive glucose, which will eventually exhaust the ability of

D

Rationale: The patient with IFG is at risk for developing type 2 diabetes, but this risk can

be decreased with lifestyle changes. 

Glycosylated hemoglobin levels will not be elevated in IFG and the Hb A1C test is not included in prediabetes testing. Elevated insulin levels do not cause the damage to blood vessels that can occur with IFG. The liver does not produce increased levels of

glucose in IFG

Page 13: NCLEX RN Practice Questions 17

the pancreas to produce insulin, and exercise will normalize glucose production.d. the onset of diabetes and the associated

cardiovascular risks can be delayed or prevented by weight loss and exercise.

A thirty five year old male has been an insulin-dependent diabetic for five years and

now is unable to urinate. Which of the following would you most likely suspect?

A: AtherosclerosisB: Diabetic nephropathyC: Autonomic neuropathyD: Somatic neuropathy

C: Autonomic neuropathy

A type 1 diabetic patient who was admitted with severe hypoglycemia and treated tells the nurse, "I did not have any of the usual

symptoms of hypoglycemia." Which question by the nurse will help identify a

possible reason for the patient's hypoglycemic unawareness?

a. "Do you use any calcium-channel blocking drugs for blood pressure?"

b. "Have you observed any recent skin changes?"

c. "Do you notice any bloating feeling after eating?"

d. "Have you noticed any painful new ulcerations or sores on your feet?"

CRationale: Hypoglycemic unawareness is caused by autonomic neuropathy, which

would also cause delayed gastric emptying. 

Calcium-channel blockers are not associated with hypoglycemic unawareness, although -adrenergic blockers can prevent patients from having symptoms of hypoglycemia. 

Skin changes can occur with diabetes, but these are not associated with autonomic neuropathy. If the patient can feel painful

areas on the feet, neuropathy has not occurred.

After having a transverse colostomy constructed for colon cancer, discharge planning for home care would include teaching about the ostomy appliance.

Information appropriate for this intervention would include:

a. Instructing the client to report redness, swelling, fever, or pain at the site to the

physician for evaluation of infection

b. Nothing can be done about the concerns of odor with the appliance.

c. Ordering appliances through the client's health care provider

d. The appliance will not be needed when traveling.

a. Instructing the client to report redness, swelling, fever, or pain at the site to the

physician for evaluation of infection

Signs and symptoms for monitoring infection at the ostomy site are a priority evaluation for clients with new ostomies.

The remaining actions are not appropriate. There are supplies avaliable for clients to help control odor that may be incurred

because of the ostomy. Although a prescription for ostomy supplies is needed,

you can order the supplies from any medical supplier. Dependent on the location and trainability of the ostomy, appliances are

almost always worn throughout the day and when traveling

After the home health nurse has taught a patient and family about how to use

glargine and regular insulin safely, which action by the patient indicates that the

teaching has been successful?

a. The patient disposes of the open insulin vials after 4 weeks.

b. The patient draws up the regular insulin in the syringe and then draws up the

glargine.c. The patient stores extra vials of both

types of insulin in the freezer until needed.d. The patient's family prefills the syringes weekly and stores them in the refrigerator.

ARationale: Insulin can be stored at room

temperature for 4 weeks. Glargine should not be mixed with other insulins or prefilled

and stored. Freezing alters the insulin molecule and should not be done.

Amitriptyline (Elavil) is prescribed for a diabetic patient with peripheral neuropathy who has burning foot pain occurring mostly

at night. Which information should the nurse include when teaching the patient about the

new medication?

a. Amitriptyline will help prevent the transmission of pain impulses to the brain.

b. Amitriptyline will improve sleep and make you less aware of nighttime pain.

c. Amitriptyline will decrease the depression caused by the pain.

d. Amitriptyline will correct some of the blood vessel changes that cause pain.

ARationale: Tricyclic antidepressants

decrease the transmission of pain impulses to the spinal cord and brain. Tricyclics also

improve sleep quality and are used for depression, but that is not the major

purpose for their use in diabetic neuropathy. The blood vessel changes that contribute to

neuropathy are not affected by tricyclics.

Page 14: NCLEX RN Practice Questions 17

An external insulin pump is prescribed for a client with diabetes mellitus and the client asks the nurse about the functioning of the pump. The nurse bases the response on the

information that the pump:

a) is timed to release programmed doses of regular or NPH insulin into the bloodstream

at specific intervals

b) continuously infuses small amounts of NPH insulin into the bloodstream while

regularly monitoring blood glucose levels

c) is surgically attached to the pancreas and infuses regular insulin into the pancreas, which in turn releases the insulin into the

bloodstream

d) gives a small continuously dose of regular insulin subcutaneously, and the client can self-administer a bolus with an additional

dose form the pump before each meal

D.) gives a small continuously dose of regular insulin subcutaneously, and the

client can self-administer a bolus with an additional dose form the pump before each

meal

An insulin pump provides a small continuous dose of regular insulin subcutaneously

throughout the day and night, and the client can self-administer a bolus with an

additional dose from the pump before each meal as needed. Regular insulin is used in an insulin pump. An external pump is not

attached surgically to the pancreas.

Blood sugar is well controlled when Hemoglobin A1C is:

a. Below 7%b. Between 12%-15%

c. Less than 180 mg/dLd. Between 90 and 130 mg/dL

a. Below 7%

A1c measures the percentage of hemoglobin that is glycated and determines

average blood glucose during the 2 to 3 months prior to testing. Used as a

diagnostic tool, A1C levels of 6.5% or higher on two tests indicate diabetes. A1C of 6% to

6.5% is considered prediabetes.

Cardiac monitoring is initiated for a patient in diabetic ketoacidosis (DKA). The nurse

recognizes that this measure is important to identify

a. electrocardiographic (ECG) changes and dysrhythmias related to hypokalemia.

b. fluid overload resulting from aggressive fluid replacement.

c. the presence of hypovolemic shock related to osmotic diuresis.

d. cardiovascular collapse resulting from the effects of hyperglycemia.

A

Rationale: The hypokalemia associated with metabolic acidosis can lead to potentially

fatal dysrhythmias such as ventricular tachycardia and ventricular fibrillation,

which would be detected with ECG monitoring. 

Fluid overload, hypovolemia, and cardiovascular collapse are possible complications of DKA, but cardiac

monitoring would not detect theses.

client newly diagnosed with diabetes mellitus has been stabilized with daily insulin injections. A nurse prepares a

discharge teaching plan regarding the insulin and plans to reinforce which of the

following concepts?

a) always keep insulin vials refrigeratedb) ketones in the urine signify a need for

less insulinc) increase the amount of insulin before

unusual exercised) systematically rotate insulin injections

within one anatomic site

d) systematically rotate insulin injections within one anatomic site

Insulin doses should not be adjusted nor increased before unusual exercise. If

ketones are found in the urine, it possibly may indicate the need for additional insulin. To minimize the discomfort associated with

insulin injections, insulin should be administered at room temperature. Injection sites should be rotated systematically within

one anatomic site.

Colon cancer is most closely associated with which of the following conditions?

a. appendicitisb. hemorroidsc. hiatal hernia

d. ulcerative colitis

d. ulcerative colitis 

Chronic ulcerative colitis, granulomas, and familial polyps seem to increase a person's

chance of developing colon cancer. The other conditions listed have no known effect

on the colon cancer risk.

During a teaching session, the nurse tells the client that 50% to 60% of daily calories

should come from carbohydrates. What should the nurse say about the types of

carbohydrates that can be eaten?

a. Simple carbohydrates are absorbed more rapidly than complex carbohydrates. 

b. Simple sugars cause a rapid spike in glucose levels and should be avoided

c. Simple sugars should never be consumed by someone with diabetes.

D

It is recommended that carbohydrates provide 50% to 60% of the daily calories.

Approximately 40% to 50% should be from complex carbohydrates. The remaining 10%

to 20% of carbohydrates could be from simple sugars. Research provides no

evidence that carbohydrates from simple sugars are digested and absorbed more rapidly than are complex carbohydrates, and they do not appear to affect blood

sugar control.

Page 15: NCLEX RN Practice Questions 17

d. Try to limit simple sugars to between 10% and 20% of daily calories.

During treatment of a patient with a Minnesota balloon tamponade for bleeding esophageal varices, which nursing action

will be included in the plan of care?

a. Encourage the patient to cough and deep breathe.

b. Insert the tube and verify its position q4hr.

c. Monitor the patient for shortness of breath.

d. Deflate the gastric balloon q8-12hr.

c. Monitor the patient for shortness of breath.

Rationale: The most common complication of balloon tamponade is aspiration

pneumonia. 

In addition, if the gastric balloon ruptures, the esophageal balloon may slip upward

and occlude the airway. 

Coughing increases the pressure on the varices and increases the risk for bleeding. 

The health care provider inserts the tube and verifies the position. The esophageal balloon is deflated every 8 to 12 hours to avoid necrosis, but if the gastric balloon is

deflated, the esophageal balloon may occlude the airway.

he goal for pre-prandial blood glucose for those with Type 1 diabetes mellitus is:

a. <80 mg/dl b. < 130 mg/dl c. <180 mg/dl 

d. <6%

b. < 130 mg/dl

Intramuscular glucagon is administered to an unresponsive patient for treatment of hypoglycemia. Which action should the

nurse take after the patient regains consciousness?

a. Give the patient a snack of cheese and crackers.

b. Have the patient drink a glass of orange juice or nonfat milk.

c. Administer a continuous infusion of 5% dextrose for 24 hours.

d. Assess the patient for symptoms of hyperglycemia.

A

Rationale: Rebound hypoglycemia can occur after glucagon administration, but having a

meal containing complex carbohydrates plus protein and fat will help prevent

hypoglycemia. 

Orange juice and nonfat milk will elevate blood sugar rapidly, but the cheese and

crackers will stabilize blood sugar. 

Administration of glucose intravenously might be used in patients who were unable

to take in nutrition orally. The patient should be assessed for symptoms of hypoglycemia

after glucagon administration.

Liver biopsy: Right side position post procedure to prevent patient from bleeding.

Mr. L. has a seven-year history of hepatic cirrhosis. He was brought to the emergency

room because he began vomiting large amounts of dark-red blood. An Esophageal Balloon Tamponade tube was inserted to

tamponade the bleeding esophageal varices. While the balloon tamponade is in place, the nurse caring for Mr. L. gives the

highest priority to

a. assessing his stools for occult blood.b. evaluating capillary refill in extremities.

c. auscultating breath sounds.d. performing frequent mouth care.

c. auscultating breath sounds.

Rationale: Airway obstruction and aspiration of gastric contents are potential serious

complications of balloon tamponade. Frequent assessment of the client's

respiratory status is the priority.

Of which of the following symptoms might an older woman with diabetes mellitus

complain?

a. Anorexiab. Pain intolerance

c. Weight lossd. Perineal itching

d. Perineal itching

One of the benefits of Glargine (Lantus) insulin is its ability to:

a. Release insulin rapidly throughout the day to help control basal glucose.

b. Release insulin evenly throughout the day and control basal glucose levels.

Glargine (Lantus) insulin is designed to release insulin evenly throughout the day

Page 16: NCLEX RN Practice Questions 17

b. Release insulin evenly throughout the day and control basal glucose levels.

c. Simplify the dosing and better control blood glucose levels during the day.

d. Cause hypoglycemia with other manifestation of other adverse reactions.

and control basal glucose levels.

Physician's orders for a client with acute pancreatitis include the following: strict

NPO, NG tube to low intermittent suction. The nurse recognizes that these

interventions will:

a. Reduce the secretion of pancreatic enzymes

b. Decrease the client's need for insulinc. Prevent secretion of gastric acidd. Eliminate the need for analgesia

a. Reduce the secretion of pancreatic enzymes

Prediabetes is associated with all of the following except:

a. Increased risk of developing type 2 diabetes

b. Impaired glucose tolerancec. Increased risk of heart disease and stroke

d. Increased risk of developing type 1 diabetes

D: Increased risk of developing type 1 diabetes

Persons with elevated glucose levels that do not yet meet the criteria for diabetes are

considered to have prediabetes and are at increased risk of developing type 2

diabetes. Weight loss and increasing physical activity can help people with

prediabetes prevent or postpone the onset of type 2 diabetes.

Proliferative retinopathy is often treated using:

a. Tonometryb. Fluorescein angiogram

c. Antibioticsd. Laser surgery

d. Laser surgery

Scatter laser treatment is used to shrink abnormal blood vessels in an effort to

preserve vision. When there is significant bleeding in the eye, it is removed in a

procedure known as vitrectomy. Tonometry is a diagnostic test that measures pressure inside the eye. A fluorescein angiogram is a diagnostic test that traces the flow of dye

through the blood vessels in the retina; it is used to detect macular edema.

Radiation therapy is used to treat colon cancer before surgery for which of the

following reasons?

a. Reducing the size of the tumorb. Eliminating the malignant cells

c. Curing the cancerd. Helping the bowel heal after surgery

a. Reducing the size of the tumor

Radiation therapy is used to treat colon cancer before surgery to reduce the size of the tumor, making it easier to be resected. 

Radiation therapy isn't curative, can't eliminate the malignant cells (though it

helps define tumor margins), can could slow postoperative healing.

Risk factors for type 2 diabetes include all of the following except:

a. Advanced ageb. Obesityc. Smoking

d. Physical inactivity

C: smoking

Additional risk factors for type 2 diabetes are a family history of diabetes,

impaired glucose metabolism, history of gestational diabetes, and race/ethnicity.

African-Americans, Hispanics/Latinos, Asian Americans, Native Hawaiians, Pacific

Islanders, and Native Americans are at greater risk of developing diabetes than

whites.

The client tells the nurse that the client really misses having sugar with tea in the morning. What is an alternative that the nurse could advise them to help sweeten

their drink. 

a. Oatrim c. sucraloseb. Olestra d. tannin

CAspartame is the generic name for a

sweetener composed of two amino acids, phenylalanine and aspartic acid. Olestra and

Oatrim are fat replacers and tannin is an acid found in some foods such as tea.

The doctor is interested in how well a client has controlled their blood glucose since

CHbgA1c is a blood test used to determine

Page 17: NCLEX RN Practice Questions 17

their last visit. What lab values could the nurse evaluate to determine how well the client has controlled their blood glucose

over the past three months?

how well blood glucose has been controlled for the last three months.

The guidelines for Carbohydrate Counting as medical nutrition therapy for diabetes

mellitus includes all of the following EXCEPT:

a. Flexibility in types and amounts of foods consumed

b. Unlimited intake of total fat, saturated fat and cholesterol

c. Including adequate servings of fruits, vegetables and the dairy group

d. Applicable to with either Type 1 or Type 2 diabetes mellitus

b. Unlimited intake of total fat, saturated fat and cholesterol

The nurse assisting in the admission of a client with diabetic ketoacidosis will

anticipate the physician ordering which of the following types of intravenous solution if

the client cannot take fluids orally?

a. 0.45% normal saline solutionb. Lactated Ringer's solutionc. 0.9 normal saline solution

d. 5% dextrose in water (D5W)

a. 0.45% normal saline solution

The nurse identifies a nursing diagnosis of risk for impaired skin integrity for a patient

with cirrhosis who has ascites and 4+ pitting edema of the feet and legs. An appropriate nursing intervention for this problem is to

a. restrict dietary protein intake.b. arrange for a pressure-relieving mattress.

c. perform passive range of motion QID.d. turn the patient every 4 hours.

B

Rationale: The pressure-relieving mattress will decrease the risk for skin breakdown for

this patient. 

Dietary protein intake may be increased in patients with ascites to improve oncotic

pressure. 

Turning the patient every 4 hours will not be adequate to maintain skin integrity. 

Passive range of motion will not take pressure off areas like the sacrum that are

vulnerable to breakdown.

The nurse identifies the collaborative problem of potential complication:

electrolyte imbalance for a patient with severe acute pancreatitis. Assessment

findings that alert the nurse to electrolyte imbalances associated with acute

pancreatitis include

a. muscle twitching and finger numbness.b. paralytic ileus and abdominal distention.

c. hypotension.d. hyperglycemia.

ARationale: Muscle twitching and finger numbness indicate hypocalcemia, a

potential complication of acute pancreatitis. 

The other data indicate other complications of acute pancreatitis but are not indicators

of electrolyte imbalance.

The nurse is caring for a client with cirrhosis of the liver. The client has developed ascites

and requires a paracentesis. Which of the following symptoms is associated with ascites and should be relieved by the

paracentesis?

A. Pruritus. B. Dyspnea. C. Jaundice. 

D. Peripheral neuropathy.

Correct answer: B

Ascites (fluid buildup in the abdomen) puts pressure on the diaphragm, resulting in

difficulty breathing and dyspnea. Paracentesis (surgical puncture of the

abdominal cavity to aspirate fluid) is done to remove fluid from the abdominal cavity and thus reduce pressure on the diaphragm in

order to relieve the dyspnea. Pruritus, jaundice, and peripheral neuropathy are signs of cirrhosis that aren't relieved or

treated by paracentesis.

The nurse is caring for a patient with a diagnosis of hypothyroidism. Which nursing diagnosis should the nurse most seriously consider when analyzing the needs of the

patient?

a. High risk for aspiration related to severe vomiting

b. Diarrhea related to increased peristalsisc. Hypothermia related to slowed metabolic

c. Hypothermia related to slowed metabolic rate

Thyroid hormone deficiency results in reduction in the metabolic rate, resulting in hypothermia, and does predispose the older

adult to a host of other health-related issues. One quarter of affected elderly

experience constipation.

Page 18: NCLEX RN Practice Questions 17

rated. Oral mucous membrane, altered related

to disease process

The nurse is caring for patients in the student health center. A patient confides to

the nurse that the patient's boyfriend informed her that he tested positive for

Hepatitis B. Which of the following responses by the nurse is BEST?

a. "That must have been a real shock to you"

b. "You should be tested for Hepatitis B"c. "You'll receive the Hepatitis B immune

globulin HBIGd. "Have you had unprotected sex with your

boyfriend"

D.

Hepatitis B is transmitted through parenteral drug abuse and sexual contact.Determine exposure before implementing.

The nurse is caring for the client diagnosed with ascites from hepatic cirrhosis. What

information should the nurse report to the health-care provider?

a. A decrease in the client's daily weight of one (1) pound.

b. An increase in urine output after administration of a diuretic.

c. An increase in abdominal girth of two (2) inches.

d. A decrease in the serum direct bilirubin to 0.6 mg/dL.

c. An increase in abdominal girth of two (2) inches.

Rationale: An increase in abdominal girth would indicate that the ascites is increasing,

meaning that the client's condition is becoming more serious and should be reported to the health-care provider.

The nurse is doing teaching with the family of a client with liver failure. Which of the following foods should the nurse advise

them to limit in the client's diet?

A. Meats and beans. B. Butter and gravies. C. Potatoes and pasta. D. Cakes and pastries.

Correct answer: A

Meats and beans are high-protein foods and are restricted with liver failure. 

In liver failure, the liver is unable to metabolize protein adequately, causing

protein by-products to build up in the body rather than be excreted. 

This causes problems such as hepatic encephalopathy (neurologic syndrome that

develops as a result of rising blood ammonia levels). Although other nutrients,

such as fat and carbohydrates, may be regulated, it's most important to limit

protein in the diet of the client with liver failure.

The nurse is educating a pregnant client who has gestational diabetes. Which of the

following statements should the nurse make to the client? Select all that apply.

a. Cakes, candies, cookies, and regular soft drinks should be avoided. 

b. Gestational diabetes increases the risk that the mother will develop diabetes later

in life.

c. Gestational diabetes usually resolves after the baby is born.

d. Insulin injections may be necessary.

e. The baby will likely be born with diabetes

f. The mother should strive to gain no more weight during the pregnancy.

ANS: A, B, C, D

Gestational diabetes can occur between the 16th and 28th week of pregnancy. 

If not responsive to diet and exercise, insulin injections may be necessary. 

Concentrated sugars should be avoided. Weight gain should continue, but not in

excessive amounts.

Usually, gestational diabetes disappears after the infant is born. However, diabetes

can develop 5 to 10 years after the pregnancy.

The nurse is having difficulty obtaining a capillary blood sample from a client's finger

to measure blood glucose using a blood glucose monitor. Which procedure will increase the blood flow to the area to

ensure an adequate specimen?

a. Raise the hand on a pillow to increase venous flow.

b. Pierce the skin with the lancet in the middle of the finger pad.

c. Wrap the finger in a warm cloth for 30-60

c. Wrap the finger in a warm cloth for 30-60 seconds.

The hand is lowered to increase venous flow. 

The finger is pierced lateral to the middle of the pad perpendicular to the skin surface.

Page 19: NCLEX RN Practice Questions 17

seconds.d. Pierce the skin at a 45-degree angle.

The nurse is performing discharge teaching for a patient with Addison's disease. It is

MOST important for the nurse to instruct the patient about:

a. signs and symptoms of infectionb. fluid and electrolyte balance

c. seizure precautionsd. steroid replacement

d. steroid replacement

steroid replacement is the most important information the client needs to know.

The nurse is planning dietary changes for a client following an episode of pancreatitis.

Which diet is suitable for the client?

a. Low calorie, low carbohydrate

b. High calorie, low fat

c. High protein, high fat

d. Low protein, high carbohydrate

b. High calorie, low fat

The nurse is working with an overweight client who has a high-stress job and smokes. This client has just received a diagnosis of Type II Diabetes and has just been started

on an oral hypoglycemic agent. Which of the following goals for the client which if met,

would be most likely to lead to an improvement in insulin efficiency to the

point the client would no longer require oral hypoglycemic agents? 

a. Comply with medication regimen 100% for 6 months

b. Quit the use of any tobacco products by the end of three months

c. Lose a pound a week until weight is in normal range for height and exercise 30

minutes daily

d. Practice relaxation techniques for at least five minutes five times a day for at least five

months

C. Lose a pound a week until weight is in normal range for height and exercise 30

minutes daily

When type II diabetics lose weight through diet and exercise they sometimes have an improvement in insulin efficiency sufficient to the degree they no longer require oral

hypoglycemic agents.

The nurse teaches the diabetic patient who rides a bicycle to work every day to administer morning insulin into the

a. thigh.b. buttock.

c. arm.d. abdomen.

D

Rationale: Patients should be taught not to administer insulin into a site that will be

exercised because exercise will increase the rate of absorption. 

The thigh, buttock, and arm are all exercised by riding a bicycle.

The nurse working in the physician's office is reviewing lab results on the clients seen

that day. One of the clients who has classic diabetic symptoms had an eight-hour

fasting plasma glucose test done. The nurse realizes that diagnostic criteria developed by the American Diabetes Association for

diabetes include classic diabetic symptoms plus which of the following fasting plasma

glucose levels?

a. Greater than 106 mg/dlb. Greater than 126 mg/dlc. Higher than 140 mg/dld. Higher than 160 mg/dl

d. Higher than 160 mg/dl

Thyroidectomy: Semi Fowler and avoid hyperflexion and hyperextension of the neck

When a client learned that the symptoms of diabetes were caused by high levels of blood glucose the client decided to stop

eating carbohydrates. In this instance, the

AWhen a client's carbohydrate consumption is inadequate ketones are produced from

Page 20: NCLEX RN Practice Questions 17

nurse would be concerned that the client would develop what complication?

a. acidosis b. atherosclerosis 

c. glycosuriad. retinopathy

the breakdown of fat. 

These ketones lower the pH of the blood, potentially causing acidosis that can lead to

a diabetic coma.

When assessing the neurologic status of a patient with a diagnosis of hepatic

encephalopathy, the nurse asks the patient to

a. stand on one foot.b. ambulate with the eyes closed.

c. extend both arms.d. perform the Valsalva maneuver.

CRationale: Extending the arms allows the

nurse to check for asterixis, a classic sign of hepatic encephalopathy. The other tests

might also be done as part of the neurologic assessment but would not be diagnostic for

hepatic encephalopathy.

When assessing the patient experiencing the onset of symptoms of type 1 diabetes,

which question should the nurse ask?

a. "Have you lost any weight lately?"b. "Do you crave fluids containing sugar?"

c. "How long have you felt anorexic?"d. "Is your urine unusually dark-colored?"

A

Rationale: Weight loss occurs because the body is no longer able to absorb glucose

and starts to break down protein and fat for energy. The patient is thirsty but does not necessarily crave sugar- containing fluids. Increased appetite is a classic symptom of type 1 diabetes. With the classic symptom

of polyuria, urine will be very dilute.

When lactulose (Cephulac) 30 ml QID is ordered for a patient with advanced

cirrhosis, the patient complains that it causes diarrhea. The nurse explains to the patient that it is still important to take the

drug because the lactulose will

a. promote fluid loss.b. prevent constipation.

c. prevent gastrointestinal (GI) bleeding.d. improve nervous system function.

d. improve nervous system function.

Rationale: The purpose for lactulose in the patient with cirrhosis is to lower ammonia

levels and prevent encephalopathy. 

Although the medication may promote fluid loss through the stool, prevent constipation,

and prevent bearing down during bowel movements (which could lead to esophageal bleeding), the medication is not ordered for

these purposes for this patient.

When obtaining a health history from a patient with acute pancreatitis, the nurse

asks the patient specifically about a history of

a. cigarette smoking.b. alcohol use.

c. diabetes mellitus.d. high-protein diet.

BRationale: Alcohol use is one of the most

common risk factors for pancreatitis in the United States. Cigarette smoking, diabetes, and high-protein diets are not risk factors.

When obtaining a health history from a patient with acute pancreatitis, the nurse

asks the patient specifically about a history of

A. smokingB. alcohol use

C. diabetes mellitusD. high-fat dietary intake

Answer: B pancreatitis is associated with alcoholism

When taking a health history, the nurse screens for manifestations suggestive of diabetes type I. Which of the following

manifestations are considered the primary manifestations of diabetes type I and would be most suggestive of diabetes type I and

require follow-up investigation? 

a. Excessive intake of calories, rapid weight gain, and difficulty losing weight

b. Poor circulation, wound healing, and leg ulcers,

c. Lack of energy, weight gain, and depression

d. An increase in three areas: thirst, intake of fluids, and hunger

D. An increase in three areas: thirst, intake of fluids, and hunger

The primary manifestations of diabetes type I are polyuria (increased urine output),

polydipsia (increased thirst), polyphagia (increased hunger).

Page 21: NCLEX RN Practice Questions 17

When working in the community, the nurse will recommend routine screening for

diabetes when the person has one or more of seven risk criteria. Which of the following

persons that the nurse comes in contact with most needs to be screened for diabetes

based on the seven risk criteria?

a. A woman who is at 90% of standard body weight after delivering an eight-pound baby

b. A middle-aged Caucasian male

c. An older client who is hypotensive

d. A client with an HDL cholesterol level of 40 mg/dl and a triglyceride level of 300

mg/dl

d. A client with an HDL cholesterol level of 40 mg/dl and a triglyceride level of 300

mg/dl

The seven risk criteria include: greater than 120% of standard body weight, Certain

races but not including Caucasian, delivery of a baby weighing more than 9 pounds or a

diagnosis of gestational diabetes, hypertensive, HDL greater than 35 mg/dl or

triglyceride level greater than 250 or a triglyceride level of greater than 250 mg/dl, and, lastly, impaired glucose tolerance or impaired fasting glucose on prior testing.

Which laboratory test should a nurse anticipate a physician would order when an

older person is identified as high-risk for diabetes mellitus? (Select all that apply.)

a. Fasting Plasma Glucose (FPG)b. Two-hour Oral Glucose Tolerance Test

(OGTT)c. Glycosylated hemoglobin (HbA1C)

d. Finger stick glucose three times daily

a. Fasting Plasma Glucose (FPG)b. Two-hour Oral Glucose Tolerance Test

(OGTT)

When an older person is identified as high-risk for diabetes, appropriate testing would include FPG and OGTT. A FPG greater than 140 mg/dL usually indicates diabetes. The

OGTT is to determine how the body responds to the ingestion of carbohydrates

in a meal. HbA1C evaluates long-term glucose control. A finger stick glucose three

times daily spot-checks blood glucose levels.

Which of the following diabetes drugs acts by decreasing the amount of glucose

produced by the liver?

a. Sulfonylureasb. Meglitinidesc. Biguanides

d. Alpha-glucosidase inhibitors

c. Biguanides

Biguanides, such as metformin, lower blood glucose by reducing the amount of glucose

produced by the liver. Sulfonylureas and Meglitinides stimulate the beta cells of the pancreas to produce more insulin. Alpha-

glucosidase inhibitors block the breakdown of starches and some sugars, which helps to

reduce blood glucose levels

Which of the following diagnostic tests should be performed annually over age 50

to screen for colon cancer?

a. Abdominal CT scanb. Abdominal x-ray

c. Colonoscopyd. Fecal occult blood test

d. Fecal occult blood test

Surface blood vessels of polyps and cancers are fragile and often bleed with the passage

of stools. 

Abdominal x-ray and CT scan can help establish tumor size and metastasis. 

A colonoscopy can help locate a tumor as well as polyps, which can be removed

before they become malignant.

Which of the following diets is most commonly associated with colon cancer?

a. low fiber, high fatb. low fat high fiber

c. low protein, high carbd. low carb, high protein

a. a. low fiber, high fat

low fiber, high fat diet reduced motility and increases the chance of constipation. The metabolic end products of this type of diet are carcinogenic. A LOW FAT HIGH FIBER diet is recommended to help avoid colon cancer. Carbohydrates and protein aren't necessarily associated with colon cancer.

Which of the following factors are risks for the development of diabetes mellitus?

(Select all that apply.)

a. Age over 45 yearsb. Overweight with a waist/hip ratio >1

c. Having a consistent HDL level above 40 mg/dl

d. Maintaining a sedentary lifestyle

a. Age over 45 yearsb. Overweight with a waist/hip ratio >1

d. Maintaining a sedentary lifestyle

Aging results in reduced ability of beta cells to respond with insulin effectively. 

Overweight with waist/hip ratio increase is part of the metabolic syndrome of DM II. 

There is an increase in atherosclerosis with DM due to the metabolic syndrome and

sedentary lifestyle.

Which of the following is accurate pertaining to physical exercise and type 1 diabetes

a. Physical exercise can slow the progression of diabetes mellitus.

Page 22: NCLEX RN Practice Questions 17

mellitus?

a. Physical exercise can slow the progression of diabetes mellitus.

b. Strenuous exercise is beneficial when the blood glucose is high.

c. Patients who take insulin and engage in strenuous physical exercise might

experience hyperglycemia.

d. Adjusting insulin regimen allows for safe participation in all forms of exercise.

Physical exercise slows the progression of diabetes mellitus, because exercise has

beneficial effects on carbohydrate metabolism and insulin sensitivity. 

Strenuous exercise can cause retinal damage, and can cause hypoglycemia. 

Insulin and foods both must be adjusted to allow safe participation in exercise.

Which of the following persons would most likely be diagnosed with diabetes mellitus?

A 44-year-old:

a. Caucasian woman.b. Asian woman.

c. African-American woman.d. Hispanic male.

c. African-American woman.

Age-specific prevalence of diagnosed diabetes mellitus (DM) is higher for African-

Americans and Hispanics than for Caucasians. Among those younger than 75,

black women had the highest incidence.

Which of the following symptoms is a client with colon cancer most likely to exhibit?

a. A change in appetiteb. A change in bowel habits

c. An increase in body weightd. An increase in body temperature

b. A change in bowel habits

The most common complaint of the client with colon cancer is a change in bowel

habits. 

The client may have anorexia, secondary abdominal distention, or weight loss.

Fever isn't associated with colon cancer.

Which of the following things must the nurse working with diabetic clients keep in mind

about Hyperglycemic Hyperosmolar Nonketotic Syndrome (HHNS)? 

A. This syndrome occurs mainly in people with Type I Diabetes

B. It has a higher mortality rate than Diabetic Ketoacidosis

C. The client with HHNS is in a state of overhydration

D. This condition develops very rapidly

B. It has a higher mortality rate than Diabetic Ketoacidosis

HHNS occurs only in people with Type II Diabetes. It is a medical emergency and has

a higher mortality rate than Diabetic Ketoacidosis. This condition develops very

slowly over hours or days.

Which of these laboratory values noted by the nurse when reviewing the chart of a diabetic patient indicates the need for

further assessment of the patient?

a. Fasting blood glucose of 130 mg/dlb. Noon blood glucose of 52 mg/dl

c. Glycosylated hemoglobin of 6.9%d. Hemoglobin A1C of 5.8%

b. Noon blood glucose of 52 mg/dl

The nurse should assess the patient with a blood glucose level of 52 mg/dl for

symptoms of hypoglycemia, and give the patient some carbohydrate-containing

beverage such as orange juice. The other values are within an acceptable range for a

diabetic patient.

Which one of the following methods/techniques will the nurse use when

giving insulin to a thin person? [Hint]

A. Pinch the skin up and use a 90 degree angle

B. Use a 45 degree angle with the skin pinched up

C. Massage the area of injection after injecting the insulin

D. Warm the skin with a warmed towel or washcloth prior to the injection

a. Pinch the skin up and use a 90 degree angle

The best angle for a thin person is 90 degrees with the skin pinched up. The area is not massaged and it is not necessary to

warm it.

While hospitalized and recovering from an episode of diabetic ketoacidosis, the patient calls the nurse and reports feeling anxious, nervous, and sweaty. Based on the patient's

report, the nurse should

A

Rationale: The patient's clinical manifestations are consistent with

hypoglycemia and the initial action should

Page 23: NCLEX RN Practice Questions 17

a. obtain a glucose reading using a finger stick.

b. administer 1 mg glucagon subcutaneously.

c. have the patient eat a candy bar.d. have the patient drink 4 ounces of orange

juice.

be to check the patient's glucose with a finger stick or order a stat blood glucose.

If the glucose is low, the patient should ingest a rapid-acting carbohydrate, such as

orange juice.

Glucagon might be given if the patient's symptoms become worse or if the patient is

unconscious. 

Candy bars contain fat, which would slow down the absorption of sugar and delay the

response to treatment.

While preparing the client for a colonoscopy, the nurse's responsibilities include:

a. Explaining the risks and benefits of the exam

b. Instructing the client about the bowel preparation prior to the test

c, Instructing the client about medication that will be used to sedate the clientd. Explaining the results of the exam

b. Instructing the client about the bowel preparation prior to the test

The nurse is responsible for instructing the client about the bowel preparation prior to

the test.Answers 1, 3, 4 are the physician's

responsibility.

You are doing some teaching with a client who is starting on a sulfonylurea antidiabetic agent. The client mentions that he usually has a couple of beers each night and takes an aspirin each day to prevent heart attack and/or strokes. Which of the following responses would be best on the part of the nurse?

a. As long as you only drink two beers and take one aspirin, this should not be a problem

b. The aspirin is alright but you need to give up drinking any alcoholic beverages

c. Aspirin and alcohol will cause the stomach to bleed more when on a sulfonylurea drug

d. Taking alcohol and/or aspirin with a sulfonylurea drug can cause development of hypoglycemia D. Taking alcohol and/or aspirin with a sulfonylurea drug can cause development of hypoglycemia

Alcohol and/or aspirin taken with a sulfonylurea can cause development of hypoglycemia.

1. Nurse Jay is performing wound care. Which of the following practices violates surgical asepsis?

A. Holding sterile objects above the waist

B.

Considering a 1″ edge around the sterile field as being contaminated

C. Pouring solution onto a sterile field cloth

D. Opening the outermost flap of a sterile package away from the body

2. During the acute phase of a burn, the nurse in-charge should assess which of the following?A.Client’s lifestyleB. Alcohol use

C.Tobacco useD.Circulatory status

3. Nurse Kate is changing a dressing and providing wound care. Which activity should she perform first?A. Assess the drainage in the dressing.

B. Slowly remove the soiled dressing

C. Wash hands thoroughly.

D.Put on latex gloves.

4. Nurse May is caring for an elderly bedridden adult. To prevent pressure ulcers, which intervention should the nurse include in the plan of care?

A.Turn and reposition the client at least once every 8 hours.

B. Vigorously massage lotion into bony prominences.

C. Post a turning schedule at the client’s bedside.

D. Slide the client, rather than lifting, when turning.

5. Nurse Jane formulates a nursing diagnosis of Impaired physical mobility for a client with third-degree burns on the lower portions of both legs. To complete the nursing diagnosis statement, the nurse should add which “related-to” phrase?

A. Related to fat emboli

B.Related to infection

C.Related to femoral artery occlusion

D.Related to circumferential eschar6. The nurse is assessing for the presence of cyanosis in a male dark-skinned client. The nurse understands that which body area would provide the best

assessment?A.LipsB. Sacrum

Page 24: NCLEX RN Practice Questions 17

C. Earlobes

D. Back of the hands

7. Which of the following individuals is least likely to be at risk of developing psoriasis?A. A 32 year-old-African American

B.A woman experiencing menopause

C. A client with a family history of the disorder

D.An individual who has experienced a significant amount of emotional distress

8. Which of the following clients would least likely be at risk of developing skin breakdown?

A.A client incontinent of urine feces

B. A client with chronic nutritional deficiencies

C.A client with decreased sensory perception

D.A client who is unable to move about and is confined to bed

9. The nurse prepares to care for a male client with acute cellulites of the lower leg. The nurse anticipates that which of the following will be prescribed for the client?

A.Cold compress to the affected area

B. Warm compress to the affected area

C.Intermittent heat lamp treatments four times daily

D. Alternating hot and cold compresses continuously

10. The clinic nurse assesses the skin of a white characteristic is associated with this skin disorder?A. Clear, thin nail beds

B.Red-purplish scaly lesions

C. Oily skin and no episodes of pruritus

D.Silvery-white scaly patches on the scalp, elbow, knees, and sacral regions

11. The clinic nurse notes that the physician has documented a diagnosis of herpes zoster (shingles) in the male client’s chart. Based on an understanding of the cause of this disorder, the nurse determines that this definitive diagnosis was made following which diagnostic test?

A.Patch testB.Skin biopsy

C.Culture of the lesion

D.Woo’s light examination

12. The nurse is assigned to care for a female client with herpes zoster (Shingles). Which of the following characteristics would the nurse expect to note when assessing the lesions of this infection?

A. Clustered skin vesicles

B. A generalized body rash

C.Small blue-white spots with a red base

D. A fiery red, edematous rash on the cheeks

13. When assessing a lesion diagnosed as malignant melanoma, the nurse in-charge most likely expects to note which of the following?A. An irregular shaped lesion

B. A small papule with a dry, rough scale

C. A firm, nodular lesion topped with crust

D.A pearly papule with a central crater and a waxy border

14. The nurse prepares discharge instructions for a male client following cryosurgery for the treatment of a malignant skin lesion. Which of the following should the nurse include in the instruction?

A.Avoid showering for 7 to 10 days

B. Apply ice to the site to prevent discomfort

C.Apply alcohol-soaked dressing twice a day

D.Clean the site with hydrogen peroxide to prevent infection

15. Nurse Carl reviews the client’s chart and notes that the physician has documented a diagnosis of paronychia. Based on this diagnosis, which of the following would the nurse expect to note during the assessment?

A.Red shiny skin around the nail bed

B.White taut skin in the popliteal area

C.White silvery patches on the elbows

D.Swelling of the skin near the parotid gland

16. A male client arrives at the emergency room and has experienced frostbites to the right hand. Which of the following would the nurse note on assessment of the client’s hand?

A. A pink, edematous hand

B.A fiery red skin with edema in the nail beds

C.Black fingertips surrounded by an erythematous rash

D.A white color to the skin, which is insensitive to touch17. The evening nurse reviews the nursing documentation in the male client’s chart and notes that the day nurse has documented that the client has a

stage II pressure ulcer in the sacral area. Which of the following would the nurse expect to note on assessment of the client’s sacral area?A.Intact skinB.Full-thickness skin loss

C.Exposed bone, tendon, or muscle

D. Partial-thickness skin loss of the dermis

18. Nurse Ivy is implementing a teaching plan to a group of adolescents regarding the causes of acne. Which of the following is an appropriate nursing

Page 25: NCLEX RN Practice Questions 17

statement regarding the cause of this disorder?A. “Acne is caused by oily skin”

B. “The actual cause is not known”

C.“Acne is caused by eating chocolate”

D.“Acne is caused as a result of exposure to heat and humidity”

19. The nurse is reviewing the health care record of a male clients scheduled to be seen at the health care clinic. The nurse determines that which of the following individuals is at the greatest risk for development of an integumentary disorder?

A.An adolescentB. An older female

C. A physical education teacher

D.An outdoor construction worker20. A male client schedule for a skin biopsy is concerned and asks the nurse how painful the procedure is. The appropriate response by the nurse is:

A.

“There is no pain associated with this procedure”

B.

“The local anesthetic may cause a burning or stinging sensation”

C."A preoperative medication will be given so you will be sleeping and will not feel any pain”

D.“There is some pain, but the physician will prescribe an opioid analgesic following the procedure”

21. The nurse is teaching a female client with a leg ulcer about tissue repair and wound healing. Which of the following statements by the client indicates effective teaching?

A.“I’ll limit my intake of protein.”

B. “I’ll make sure that the bandage is wrapped tightly.”

C. “My foot should feel cold.”

D.“I’ll eat plenty of fruits and vegetables.”

22. Following a full-thickness (third-degree) burn of his left arm, a male client is treated with artificial skin. The client understands postoperative care of artificial skin when he states that during the first 7 days after the procedure, he will restrict:

A. range of motion.

B. protein intake

C.going outdoors.

D. fluid ingestion

23. Following a small-bowel resection, a male client develops fever and anemia. The surface surrounding the surgical wound is warm to the touch and necrotizing fasciitis is suspected. Another manifestation that would most suggest necrotizing fasciitis is:

A. erythema

B. leukocytosis

C. pressurelike pain.

D. swelling

24. While in a skilled nursing facility, a female client contracted scabies, which is diagnosed the day after discharge. The client is living at her daughter’s home, where six other persons are living. During her visit to the clinic, she asks a staff nurse, “What should my family do?” The most accurate response from the nurse is:

A.“All family members will need to be treated.”

B.

“If someone develops symptoms, tell him to see a physician right away.”

C.“Just be careful not to share linens and towels with family members.”

D.

“After you’re treated, family members won’t be at risk for contracting scabies.”

25. The nurse is assessing a male client admitted with second- and third-degree burns on the face, arms, and chest. Which finding indicates a potential problem?

A.Partial pressure of arterial oxygen (PaO2) value of 80 mm Hg

B.Urine output of 20 ml/hourC.White pulmonary secretionsD. Rectal temperature of 100.6° F (38° C)

26. A female client exhibits s purplish bruise to the skin after a fall. The nurse would document this finding most accurately using which of the following terms?

A. Purpura

B.PetechiaeC.EcchymosisD.Erythema

27. An older client’s physical examination reveals the presence of a number of bright red-colored lesions scattered on the trunk and tights. The nurse interprets that this indicates which of the following lesions due to alterations in blood vessels of the skin?

A.Cherry angiomaB. Spider angioma

C.Venous starD. Purpura

28. A nurse is reviewing the medical record of a male client to be admitted to the nursing unit and notes documentation of reticular skin lesions. The nurse expects that these lesions will appear to be:

A.Ring-shaped

B. Linear

C. Shaped like an arc

D. Net-like appearance

29. A male client seen in an ambulatory clinic has a butterfly rash across the nose. The nurse interprets that this finding is consistent with early manifestations of which of the following disorders?

A. Hyperthyroidism

B. Perncious anemia

Page 26: NCLEX RN Practice Questions 17

C.Cardiopulmonary disorders

D.Systemic lupus erythematosus (SLE)30. A female client with cellulites of the lower leg has had cultures done on the affected area. The nurse reading the culture report understands that which

of the following organisms is not part of the normal flora of the skin?A. Staphylococcus epidermidis

B. Staphylococcus aureus

C.Escherichia coli (E. coli)D. Candida albicans